Limit of sequence n^p/e^n as n approaches infinity

Click For Summary
SUMMARY

The limit of the sequence n^p/e^n as n approaches infinity is definitively 0 for any positive integer p. The derivation utilizes L'Hôpital's rule, where repeated differentiation of the numerator n^p results in p! while the denominator remains e^n. This leads to the conclusion that as n approaches infinity, the limit converges to 0. Additionally, for a more general case, rewriting n^p as e^(p ln(n)) allows for analysis of the limit of (p ln(n) - n) as n approaches infinity, confirming the result.

PREREQUISITES
  • Understanding of limits in calculus
  • Familiarity with L'Hôpital's rule
  • Knowledge of exponential functions and logarithms
  • Basic differentiation techniques
NEXT STEPS
  • Study the application of L'Hôpital's rule in various limit problems
  • Explore the properties of exponential growth versus polynomial growth
  • Learn about the behavior of logarithmic functions as n approaches infinity
  • Investigate the generalization of limits involving sequences and series
USEFUL FOR

Students and professionals in mathematics, particularly those studying calculus and analysis, as well as educators seeking to clarify concepts related to limits and derivatives.

alex_dc1
Messages
12
Reaction score
0
Simple question just as the title says, but I can't remember or derive the solution for the life of me. I know that the answer is 0. I know why the answer is 0. But I need to know the mathematical derivation of the solution, and that's the part that I can't remember. So, to reiterate, how do you find:

lim (n^p)/(e^n) as n→∞, where p>0
 
Physics news on Phys.org
Wait, I think I just figured it out, but I want to make sure this makes sense:

(n^p)/(e^n) =

n/e^(n/p) * n/e^(n/p) * n/e^(n/p) * n/e^(n/p)...

where:

lim n/e^(n/p) as n→∞ is (by L'hopital's rule) equal to:

*edit for the correct derivative:
lim 1/[ (1/p) * e^(n/p) ] n →∞ is equal to 0, therefore:

n/e^(n/p) * n/e^(n/p) * n/e^(n/p) * n/e^(n/p)... is equal to 0 * 0 * 0 * 0... therefore,

lim (n^p)/(e^n) as n→∞ is equal to 0

Does that make sense, or am I just making crap up?
 
Last edited:
Um, p-iterations of L'Hospital's rule would be much clearer, I think. The limit is certainly 0, but I think you are making it more complicated than it needs to be.
 
I've never heard that term before. Would that basically state that the p-th derivative of n^p equals (p!)?
 
alex_dc1 said:
Wait, I think I just figured it out, but I want to make sure this makes sense:

(n^p)/(e^n) =

n/e^(n/p) * n/e^(n/p) * n/e^(n/p) * n/e^(n/p)...


where:

lim n/e^(n/p) as n→∞ is (by L'hopital's rule) equal to:

*edit for the correct derivative:
lim 1/[ (1/p) * e^(n/p) ] n →∞ is equal to 0, therefore:

n/e^(n/p) * n/e^(n/p) * n/e^(n/p) * n/e^(n/p)... is equal to 0 * 0 * 0 * 0... therefore,

lim (n^p)/(e^n) as n→∞ is equal to 0

Does that make sense, or am I just making crap up?

That reminds me of the "proof" that the derivative of x^2 is just "x". Since x^n is just x added to itself n times, x^x is x added to itself x times: x^x= (x+ x+ x+ ...+ x) and therefore the derivative is (x^x)'= (1+ 1+ 1+ ...+ 1) x times which is equal to x.

That's wrong because you cannot treat doing an operation a variable number of times the same as doing it a constant number of times.

lineintegral is suggesting that you use L'Hopital's rule p times (that's what "p iteration" means). Each time you differentiate the numerator you get a lower power of nwhile the denominator is always e^n. After differentiating p times you have just p! in the numerator and e^n in the denominator. Now take the limit as n goes to infinity.

Strictly speaking, that only works for p a positive integer. For the more general case, write n^p as e^(ln(n^p)= e^(p ln(n)) so that n^p/e^n= e^(pln(n)- n) and look at the limit of pln(n)- n as n goes to infinity.
 
Question: A clock's minute hand has length 4 and its hour hand has length 3. What is the distance between the tips at the moment when it is increasing most rapidly?(Putnam Exam Question) Answer: Making assumption that both the hands moves at constant angular velocities, the answer is ## \sqrt{7} .## But don't you think this assumption is somewhat doubtful and wrong?

Similar threads

  • · Replies 7 ·
Replies
7
Views
2K
  • · Replies 20 ·
Replies
20
Views
2K
  • · Replies 1 ·
Replies
1
Views
2K
  • · Replies 24 ·
Replies
24
Views
2K
  • · Replies 14 ·
Replies
14
Views
2K
  • · Replies 23 ·
Replies
23
Views
2K
  • · Replies 6 ·
Replies
6
Views
2K
  • · Replies 8 ·
Replies
8
Views
2K
Replies
17
Views
2K
  • · Replies 4 ·
Replies
4
Views
2K